LSAT and Law School Admissions Forum

Get expert LSAT preparation and law school admissions advice from PowerScore Test Preparation.

 Administrator
PowerScore Staff
  • PowerScore Staff
  • Posts: 8916
  • Joined: Feb 02, 2011
|
#40318
Complete Question Explanation
(The complete setup for this game can be found here: lsat/viewtopic.php?t=6748)

The correct answer choice is (A)

This Justify question asks us to identify a condition that, if true, would fully determine the order of the reports. Questions of this type can be particularly challenging if you have not previously examined the variables (and rules) that are especially likely to restrict the solution of the game.

Given our discussion of the last rule, we already know that the game is left with only one solution if I is the last report in either segment. This prephrase immediately reveals that answer choice (A) is correct. If I is the last report in the first segment, then S must be the last report in the second segment in compliance with the fifth rule in the game:
PT72_Game_#1_#5_diagram 1.png
However, since I is always longer than W, we need to ensure that W is not in the same segment as I, forcing W to be the first report in the second segment:
PT72_Game_#1_#5_diagram 2.png
Finally, we must ensure that N is always the longest of the five reports (fourth rule). Since the second segment is fully occupied, N must be the first report in the first segment, leaving T to be the second report in that segment:
PT72_Game_#1_#5_diagram 3.png
If you did not have the requisite hypothetical set-up in advance, you would be forced to test each answer choice separately in order to identify which condition leaves the game with only one possible solution. This approach is typically less efficient than the alternative, but here you were in luck: the first answer choice you probably set up happened to be correct. However, what if the correct answer choice were, say, (E) instead of (A)? Do you really need to make five local setups in order to determine the correct answer choice? There is almost always a more efficient way to proceed.

Here’s an example of a question in which Templates—even if partial—would have been particularly useful. The templates reveal that the game is not particularly restricted if N happens to be the first report in either segment: there are multiple solutions in either case. This immediately eliminates answer choices (B) and (C).

Templates II and IV further show that if S is the last report in the second segment, N could be the first report in either segment. This is sufficient to eliminate answer choice (D) from consideration:
PT72_Game_#1_#5_diagram 4.png
PT72_Game_#1_#5_diagram 5.png
Finally, if W is the last report in the first segment, S must be the last report in the second segment. According to Templates II and IV, N could still be the first report in either segment—leaving the game with at least two (actually, four) possible solutions:
PT72_Game_#1_#5_diagram 6.png
PT72_Game_#1_#5_diagram 7.png
You do not have the required permissions to view the files attached to this post.

Get the most out of your LSAT Prep Plus subscription.

Analyze and track your performance with our Testing and Analytics Package.